Difference between revisions of "1986 AJHSME Problems/Problem 4"

(Solution)
m (Solution)
Line 7: Line 7:
 
==Solution==
 
==Solution==
  
The easiest way to do this problem is to estimate, since it says "closest to", and the multiple choices are pretty spread out.
+
<center>
 +
<math>(1.8)(40.37)\approx (1.8)(40)=72.</math>
 +
</center>
  
<math>1.8</math> is about <math>2</math>, <math>40.3</math> is about <math>40</math>, and <math>.07</math> is about <math>0</math>.
+
<math>74</math> is the closest to <math>72</math>, so the answer is <math>\boxed{\text{C}}</math>.
Putting this in, we get <math>(2)(40 + 0) = 80</math>
 
 
 
<math>80</math> is about <math>84</math>
 
 
 
<math>\boxed{\text{D}}</math>
 
 
 
edit: the answer is C. The reason for this is that 1.8 and 2 are close, but when multiplied by 40, it's a big difference (80 and 72). So you could just do 1.8(40.37) which is basically 1.8*40=72. 74 is the closest to 72.
 
  
 
==See Also==
 
==See Also==

Revision as of 15:40, 23 May 2010

Problem

The product $(1.8)(40.3+.07)$ is closest to

$\text{(A)}\ 7 \qquad \text{(B)}\ 42 \qquad \text{(C)}\ 74 \qquad \text{(D)}\ 84 \qquad \text{(E)}\ 737$

Solution

$(1.8)(40.37)\approx (1.8)(40)=72.$

$74$ is the closest to $72$, so the answer is $\boxed{\text{C}}$.

See Also

1986 AJHSME (ProblemsAnswer KeyResources)
Preceded by
Problem 3
Followed by
Problem 5
1 2 3 4 5 6 7 8 9 10 11 12 13 14 15 16 17 18 19 20 21 22 23 24 25
All AJHSME/AMC 8 Problems and Solutions